Đến nội dung

Hình ảnh

Tuần 2 tháng 12/2016 : Bài toán nội tiếp trên đường tròn tiếp xúc

hình học

  • Please log in to reply
Chủ đề này có 4 trả lời

#1
baopbc

baopbc

    Himura Kenshin

  • Thành viên nổi bật 2016
  • 410 Bài viết

Như vậy thầy Hùng đã đưa ra lời giải bài cũ trong tuần 2 tháng 12 và kèm theo đó là bài toán mới, xin trích dẫn lại bài toán mới,

 

Cho tam giác $ABC$ có đường tròn nội tiếp $(I)$ tiếp xúc $BC$, $CA$, $AB$ lần lượt tại $D$, $E$, $F$. $M$, $N$, $P$, $Q$ lần lượt là trung điểm của $DE$, $DF$, $EM$, $FN$. $BM$, $CN$ theo thứ tự cắt $PQ$ tại $S$, $T$. Chứng minh rằng bốn điểm $B$, $C$, $S$, $T$ cùng nằm trên một đường tròn tiếp xúc $(I)$.

Post 370.PNG

Hình vẽ bài toán

 



#2
Ngockhanh99k48

Ngockhanh99k48

    Trung sĩ

  • Thành viên
  • 127 Bài viết
Lời giải:
$ID^2=\overline{IB}.\overline{IN}=\overline{IC}.\overline{IM}$. Do đó $B, N, M, C \in (J)$. $X=MN \cap BC, Y=BM \cap CN$. Dễ thấy $(DMN)$ tiếp xúc $(I)$ nên $\overline{XB}.\overline{XC}=\overline{XM}.\overline{XN}=XD^2$ nên $X$ thuộc trục đẳng phương của $(I)$ và $(J)$. Theo định lí Brocard ta có $XY \perp IJ$ và $IX \perp JY$ tại $U$. Hơn nữa ta cũng có $U \in (BIC)$ nên $\overline{XU}.\overline{XI}=\overline{XB}.\overline{XC}=XD^2$ suy ra $\widehat{XUD}=\widehat{XDI}=90^{\circ}$. Do đó $D \in JY$.
$XY \perp IJ$ nên $Y$ thuộc trục đẳng phương của $(I)$ và $(J)$. Nếu $DY$ cắt $(I)$ tại $V$ thì dễ thấy $XV$ tiếp xúc $(BVC)$ ($VD$ là đối cực của $X$ trong $(I)$), $(I)$ và $V \in (BDM), (CDN)$. Gọi $S'=VE \cap BM, T'=VF \cap CN$.
Ta chỉ cần chứng minh $S', T' \in (BVC)$ và $S'T' \equiv PQ$ là xong. Ta có $VB, VN$ đẳng giác trong góc $\widehat{FVD}$ nên $\widehat{T'VB}=\widehat{DVN}=\widehat{BCT'}$, suy ra $T' \in (BVC)$. Tương tự $S' \in (BVC)$.
$\widehat{MES'}=\widehat{VFN}, \widehat{EMS'}=\widehat{BMD}=\widehat{BVD}=\widehat{FVN}$ nên $\triangle MES' \sim \triangle VFN$. Do đó $\frac{ES'}{EP}=\frac{2ES'}{EM}=\frac{2FN}{FV}=\frac{FD}{FV}$. Suy ra $\triangle FDV \sim \triangle ES'P$. Do đó $\widehat{FEV}=\widehat{FDV}=\widehat{ES'P}$, suy ra $PS' \parallel EF$ hay $S' \in PQ$ do $PQ \parallel EF$. Tương tự $T' \in PQ$. Ta có đpcm.

Bài viết đã được chỉnh sửa nội dung bởi Ngockhanh99k48: 12-12-2016 - 18:25


#3
baopbc

baopbc

    Himura Kenshin

  • Thành viên nổi bật 2016
  • 410 Bài viết

Một lời giải khác :

 

Theo hệ thức lượng trong tam giác vuông, $\overline{IN}\cdot \overline{IB}=ID^2=\overline{IM}\cdot \overline{IC}$ nên tứ giác $BNMC$ nội tiếp.

Mặt khác, do $MN\parallel ST$ nên $\widehat{SBC}=\widehat{MNC}=\widehat{STC}$ suy ra tứ giác $BCST$ nội tiếp.$\qquad (1)$

 

Từ $(1)$ suy ra $\widehat{PSM}=\widehat{NCD}$, $\widehat{SPM}=\widehat{CDN}$ nên $\triangle CDN\sim \triangle SPM$ (góc - góc). Do đó :

\[\frac{SM}{CN}=\frac{PM}{DN}\Rightarrow \frac{2SM}{CN}=\frac{DM}{DN}\qquad (2)\]

Gọi $K$ là giao điểm của $DM$ với đường tròn ngoại tiếp tam giác $NDC$.

Do $\widehat{NKC}=\widehat{NDB}=\widehat{NMN}$, $\widehat{KCN}=\widehat{MDN}$ nên $\triangle MDN\sim \triangle KCN$ (góc - góc). Do đó :

\[\frac{KC}{CN}=\frac{DM}{DN}\qquad (3)\]

Từ $(2)$ và $(3)$ ta suy ra $KC=2SM$ $\qquad (4)$

Gọi $G$, $H$ lần lượt là giao điểm của $CN$ với $DM$, $BM$ với $DN$.

Do $\widehat{MGN}=90^\circ-\widehat{NCM}=90^\circ-\widehat{MBN}=\widehat{MHN}$ nên tứ giác $MGHN$ nội tiếp. $\qquad (5)$

Từ $(5)$ suy ra $\widehat{SMK}=\widehat{GMH}=\widehat{DNG}=\widehat{MKC}$ $\qquad (6)$

Do tam giác $KMC$ vuông tại $M$ nên từ $(6)$, $MS$ đi qua trung điểm $CK$. Từ đó theo $(4)$, $S$ là trung điểm $CK$. $\qquad (7)$

 

Gọi $J$ là giao điểm của đường tròn ngoại tiếp tứ giác $BTSC$ với $AC$.

Từ $(7)$ suy ra $SM=SC$ hay $\triangle MSC$ cân tại $S$. Do đó $\widehat{SCJ}=\widehat{SMC}-\widehat{MCD}=\widehat{SBC}$.

Suy ra $DM$ là phân giác $\angle JBC$ hay $M$ là tâm đường tròn nội tiếp tam giác $BJC$.

Theo định lý Lyness đảo, đường tròn $(I)$ tiếp xúc đường tròn ngoại tiếp tứ giác $BCST$.


Bài viết đã được chỉnh sửa nội dung bởi baopbc: 14-12-2016 - 02:21


#4
SonKHTN1619

SonKHTN1619

    Binh nhất

  • Thành viên mới
  • 22 Bài viết

Em xin đề xuất một hướng nhìn khác của bài toán.

Cho tam giác ABC, (J) là đường tròn bàng tiếp góc A của tam giác ABC. (J) tiếp xúc BC,CA,AB tại D,E,F. M,N,P,Q lần lượt là trung điểm DE, DF, EM ,FN. PQ cắt BM, CN tại S,T. Chứng minh rằng B,C,S,T thuộc cùng một đường tròn tiếp xúc (J).

Chứng minh bài toán này hoàn toàn tương tự bài toán gốc.

Sau đây là chứng minh của em cho bài toán gốc.

Hình gửi kèm

  • Screenshot from 2016-12-16 17:42:54.png
  • IMG_0535.JPG
  • IMG_0536.JPG

Bài viết đã được chỉnh sửa nội dung bởi SonKHTN1619: 17-12-2016 - 23:15

HSGS in my heart  :icon12:


#5
quanghung86

quanghung86

    Thiếu úy

  • Điều hành viên
  • 632 Bài viết

Việc viết lại bài toán cho tâm bàng tiếp thì không mới, vì hầu như tất cả những bài toán đúng với tâm nội tiếp thì cũng đúng với tâm bàng tiếp thôi, đó cũng chưa coi là mở rộng được.







Được gắn nhãn với một hoặc nhiều trong số những từ khóa sau: hình học

2 người đang xem chủ đề

0 thành viên, 2 khách, 0 thành viên ẩn danh